36-я Международная Математическая Oлимпиада
Канада, Торонто, 1995 год


Пусть $a$, $b$, $c$ — положительные числа такие, что $abc=1$. Доказать, что $\dfrac{1}{{{a}^{3}}\left( b+c \right)}+\dfrac{1}{{{b}^{3}}\left( c+a \right)}+\dfrac{1}{{{c}^{3}}\left( a+b \right)}\ge \dfrac{3}{2}.$
посмотреть в олимпиаде

Комментарий/решение:

  2
2016-10-23 19:17:20.0 #

$$\frac{b^2\cdot c^2}{ab+ac}+\frac{a^2\cdot c^2}{bc+ab}+\frac{a^2\cdot b^2}{ac+bc}\geq \frac{3}{2}$$

$$ b \cdot c =x, a \cdot c = y, a \cdot b= z \Rightarrow $$

$$\Rightarrow \frac{x^2}{z+y}+\frac{y^2}{x+z}+\frac{z^2}{x+y}\geq \frac{3}{2}$$

$$\frac{x^2}{z+y}+\frac{y^2}{x+z}+\frac{z^2}{x+y}=\frac{x(x+y+z-y-z)}{z+y}+\frac{y(y+z+x-z-x)}{x+z}+\frac{z(z+x+y-x-y)}{x+y}=$$

$$= (x+y+z)(\frac{x}{y+z}+\frac{y}{x+z}+\frac{z}{x+y})-(x+y+z)= \frac{3}{2}(x+y+z)-(x+y+z)= \frac{x+y+z}{2}\geq \frac{3}{2}$$

пред. Правка 3   4
2021-02-15 21:47:41.0 #

Пусть $a=\dfrac 1 x,b=\dfrac 1 y,c=\dfrac 1 z\implies xyz=1.$ Тогда $$\mathrm{(!)}\quad \dfrac{x^2}{y+z}+\dfrac{y^2}{z+x}+\dfrac{z^2}{x+y}\ge\dfrac 3 2 $$

Из КБШ и AM-GM :$$\small{\dfrac{x^2}{y+z}+\dfrac{y^2}{z+x}+\dfrac{z^2}{x+y}\ge\dfrac {(x+y+z)^2}{2(x+y+z)}=\dfrac{x+y+z}{2}\ge\dfrac{ 3(xyz)^{\frac 1 3}}{2}=\dfrac 3 2.}\quad\square$$

  1
2021-02-15 19:35:16.0 #

$abc=1$ болғандықтан теңсіздіктің сол жағын $(abc)^2$-қа көбейтеміз:$$\frac{1}{a^3(b+c)}+\frac{1}{b^3(c+a)}+\frac{1}{c^3(a+b)}=\frac{(abc)^2}{a^3(b+c)}+\frac{(abc)^2}{b^3(c+a)}+\frac{(abc)^2}{c^3(a+b)}=\frac{(bc)^2}{a(b+c)}+\frac{(ac)^2}{b(c+a)}+\frac{(ab)^2}{c(a+b)}=\frac{(bc)^2}{ab+ac}+\frac{(ac)^2}{bc+ba}+\frac{(ab)^2}{ca+cb}\geq \frac{(ab+bc+ac)^2}{2(ab+bc+ac)}=\frac{ab+bc+ac}{2}.$$ Енді $\frac{ab+bc+ac}{2}\geq \frac{3}{2}$ теңсіздігін дәлелдейміз. Коши теңсіздігін қолдансақ: $ab+bc+ac\geq 3\cdot \sqrt{ab\cdot bc\cdot ca}=3abc=3\cdot 1=3.$

  0
2023-12-11 17:34:26.0 #

По AM-GM имеем три неравенства:

$$\dfrac{1}{a^3(b+c)}+\dfrac{a(b+c)}{4}\geq \dfrac{1}{a}$$

$$\dfrac{1}{b^3(a+c)}+\dfrac{b(a+c)}{4}\geq \dfrac{1}{b}$$

$$\dfrac{1}{c^3(a+b)}+\dfrac{c(a+b)}{4}\geq \dfrac{1}{c}$$

Складывая их вместе и учитывая, что $ab=\frac{1}{c}$ и т.д. получаем

$$\dfrac{1}{a^3(b+c)}+\dfrac{1}{c^3(a+b)}+\dfrac{1}{b^3(b+c)}\geq \dfrac{1}{2}(\dfrac{1}{a}+\dfrac{1}{b}+\dfrac{1}{c})$$

При этом по неравенству GM-HM, имеем

$$1=\sqrt[3]{abc}\geq \dfrac{3}{\frac{1}{a}+\frac{1}{b}+\frac{1}{c}}\Longrightarrow \dfrac{1}{a}+\dfrac{1}{b}+\dfrac{1}{c}\geq 3$$

Отсюда и выходит требуемое. Случай равенства достигает тогда и только тогда, когда $a=b=c=1$.

  4
2024-01-19 18:45:01.0 #

По КБШ:

$(\frac{1}{a^3(b+c)}$+$\frac{1}{b^3(a+c)}$+$\frac{1}{c^3(a+b)})$$(a(b+c)+b(a+c)+c(a+b))$ $\geq$ ($\frac{1}{a}$+$\frac{1}{b}$+$\frac{1}{c}$)^2

($\frac{1}{a}$+$\frac{1}{b}$+$\frac{1}{c}$)^2 $\geq$ $3(ab+bc+ca)$ тк $\frac{1}{a}$=bc и $ab+bc+ca$ $\geq$ $3$

$(\frac{1}{a^3(b+c)}$+$\frac{1}{b^3(a+c)}$+$\frac{1}{c^3(a+b)})$ $\geq$ $\frac{3(ab+bc+ca)}{2(ab+bc+ca)}$ что доказывает.

пред. Правка 3   1
2024-01-30 20:31:40.0 #

Прекрасное решение.